4

I can't figure out how to solve the infinite sum of

$\sum ^{\infty }_{n=0}\left[( -1)^{n}\right]$

I know that Srinivasa Ramanujan solved it and I couldn't figure it out with Ramanujan's summation. Here's what I tried.

Note: I'm only interested in solving this divergent sum by means of Ramanujan's summation. I know about the other methods, but am unable to solve it with Ramanujan's summation. So here's what I tried

$ \begin{array}{l} \sum ^{\infty }_{n=1}\left[( -1)^{n}\right]\overset{\Re }{=} -\frac{1}{2} +i\int\limits ^{\infty }_{0}\frac{e^{-\pi x} -e^{\pi x}}{e^{2\pi x} -1} dx=-\frac{1}{2} -i\int\limits ^{\infty }_{0}\frac{e^{-\pi x}\left( e^{2\pi x} -1\right)}{e^{2\pi x} -1} dx=-\frac{1}{2} -\frac{i}{\pi }\\ \Longrightarrow \\ \sum ^{\infty }_{n=0}\left[( -1)^{n}\right] =1-\frac{1}{2} -\frac{i}{\pi } =\frac{1}{2} -\frac{i}{\pi } \end{array}$

Which is obviously wrong.

Here are some ideas that were tried though

$\sum ^{\infty }_{n=0}\left[( -1)^{n}\right]$ Has no imaginary part--therefore

$\sum ^{\infty }_{n=0}\left[( -1)^{n}\right] =\frac{1}{2}$

The other idea is

$ \begin{array}{l} \sum\limits ^{\infty }_{n=1} f( n)\overset{\Re }{=} -\frac{f( 0)}{2} +i\int\limits ^{\infty }_{0}\frac{f( ix) -f( -ix)}{e^{2\pi x} -1} dx\\ \\ \ \ \ \ \ \ \ \ \ \ \ \ \ \ \ \ \ \ \ \ \ \ \mathfrak{Grandi's\ series}\\ \sum ^{\infty }_{n=0}( -1)^{n} =1+\sum\limits ^{\infty }_{n=1}( -1)^{n}\overset{\Re }{=}\\ 1-\frac{1}{2} +i\int\limits ^{\infty }_{0}\frac{( -1)^{ix} -( -1)^{-ix}}{e^{2\pi x} -1} dx=\frac{1}{2} +i\int\limits ^{\infty }_{0}\frac{( -1)^{ix} -( -1)^{-ix}}{e^{2\pi x} -1} dx\\ \ ( -1)^{a} =( -1)^{-a} \Longrightarrow \\ \frac{1}{2} +i\int\limits ^{\infty }_{0}\frac{( -1)^{ix} -( -1)^{ix}}{e^{2\pi x} -1} dx=\frac{1}{2} +i\int\limits ^{\infty }_{0} 0dx=\frac{1}{2} \Re \end{array}$

But this shouldn't work because the thing $( -1)^{a} =( -1)^{-a} ,\ a\in \mathbb{Z}$

And $x$ is not an integer , Right? Well, I can't figure this out. Any help is appreciated! Thanks!

Edit:

Actually, I realize that Srinivasa Ramanujan probably did it using the derichlet eta function$\Longrightarrow$

$ \begin{array}{l} \sum ^{\infty }_{n=0}( -1)^{n} =1+\sum ^{\infty }_{n=1}( -1)^{n} =1-\eta ( 0) ,\ \eta ( 0) =-\zeta ( 0) =\frac{1}{2}\\ \boxed{\therefore \sum ^{\infty }_{n=0}( -1)^{n} =\frac{1}{2}} \end{array}$

  • 1
    Where are you getting this version of the definition of Ramanujan summation from? Is it a simplification that only applies to convergent series? What growth conditions does it place on your interpolating function $f$, and have you verified that $f$ meets those conditions? – Chris Culter Aug 19 '19 at 20:10
  • The version of Ramanujan's summation that I have is from Wikipedia. Grandi's series is divergent. And I don't understand the rest of your questions. – Highvoltagemath Aug 19 '19 at 20:22
  • Is it from the paragraph that begins, "The convergent version of summation..."? – Chris Culter Aug 19 '19 at 20:23
  • It looks like it yes. – Highvoltagemath Aug 19 '19 at 20:25
  • 1
    Yeah, then it's probably not going to work here. – Chris Culter Aug 19 '19 at 20:25
  • 1
    So, when does it work? What conditions need to be met? And is there another version that Srinivasa Ramanujan used when he did it? Thanks! – Highvoltagemath Aug 19 '19 at 20:26
  • I'm afraid I don't know the answers to those questions off the top of my head, sorry! All I can comment on is why the above attempts ran into trouble. – Chris Culter Aug 19 '19 at 20:30
  • Okay Thanks so much for what you have answered! – Highvoltagemath Aug 19 '19 at 20:37
  • "Cesaro summation" could be an alternative to "Ramanujan summation". See as well https://math.stackexchange.com/q/2059629/305862 – Jean Marie Aug 19 '19 at 21:46
  • 1
    Yes, but I want to know how Ramanujan solved it. Or at least what the requirements are for the Ramanujan summation. – Highvoltagemath Aug 20 '19 at 00:21
  • I found following video: Grandi's Series | Grandi's Series Proof and i can not find mistakes there. Then i double checked in wolfram alpha, which gives result i/π. What could be happening here? – blindProgrammer Sep 16 '21 at 22:34
  • 1
    @blindProgrammer In the video he assumes that the $(-1)^n$ function has no imaginary component. This is however the correct assumption, but you have to instead use $cos(\pi \cdot n)$. Because if you have $(-1)^{(1/2)}=i$, and $(-1)^{(-1/2)}=-i$ so that negative in the exponent is really important as it will determine the sign of the imaginary part. – Highvoltagemath Sep 19 '21 at 02:54

2 Answers2

-1

$S = \sum ^{\infty }_{n=0}\left[( -1)^{n}\right] = 1 -\sum ^{\infty }_{n=0}\left[( -1)^{n}\right]=1-S$ and so we have $S=1-S \implies S = \frac{1}{2}$.

CyclotomicField
  • 11,018
  • 1
  • 12
  • 29
-1

Instead of writing the terms as $f(n)=(-1)^n$ try using just the real part $f(n)=\cos(\pi n)$.

Edit: Let me elaborate. Ramanujan uses the Euler–Maclaurin formula in the form $$ \sum_{m=1}^n f(m) \sim c + \int_a^n f(x)\,dx+\tfrac{1}{2}f(n)+\sum_{r=1}^\infty \frac{B_{2r}}{(2r)!}f^{(2r-1)}(n) $$ but if you set $f(n)=(-1)^n$ then the infinite series on the right is purely imaginary while the finite sum on the left is purely real—that is why the constant $c$ that you obtained had an imaginary component. This problem is circumvented if you choose $f$ to be purely real rather than a complex-valued function.

Daniel Fischer
  • 206,697
cbright
  • 57